\documentclass[a5paper,10pt]{article} \usepackage{myXsim} \usepackage{tasks} % Title Page \title{DM2 \hfill BOUALIA Bilel} \tribe{TST} \date{\hfillÀ render pour le Mercredi 24 février} \xsimsetup{ solution/print = false } \begin{document} \maketitle \begin{exercise}[subtitle={Loi binomiale}] Trois personnes s'apprêtent à passer le portique de sécurité. On suppose que pour chaque personne la probabilité que le portique sonne est égale à $0.68$. Soit $X$ la variable aléatoire donnant le nombre de personnes faisant sonner le portique, parmi les 3 personnes de ce groupe. \begin{enumerate} \item Tracer l'arbre représentant le situation. \item Justifier que $X$ suit une loi binomiale dont on précisera les paramètres. \item Quelle est la probabilité qu'une seule personne fasse sonner le portique? \item Calculer puis interpréter les probabilités suivantes \[ P(X = 0) \qquad \qquad P(X \geq 2) \] \item Calculer l'espérance de $X$ et interpréter le résultat. \end{enumerate} \end{exercise} \begin{solution} \begin{enumerate} \item \begin{tikzpicture}[sloped] \node {.} child {node {$0$} child {node {$0$} child {node {$0$} edge from parent node[above] {0.32} } child {node {$1$} edge from parent node[above] {0.68} } edge from parent node[above] {0.32} } child[missing] {} child {node {$1$} child {node {$0$} edge from parent node[above] {0.32} } child {node {$1$} edge from parent node[above] {0.68} } edge from parent node[above] {0.32} } edge from parent node[above] {0.32} } child[missing] {} child[missing] {} child[missing] {} child { node {$1$} child {node {$0$} child {node {$0$} edge from parent node[above] {0.32} } child {node {$1$} edge from parent node[above] {0.68} } edge from parent node[above] {0.32} } child[missing] {} child {node {$1$} child {node {$0$} edge from parent node[above] {0.32} } child {node {$1$} edge from parent node[above] {0.68} } edge from parent node[above] {0.32} } edge from parent node[above] {0.68} } ; \end{tikzpicture} \item Chaque personne a 2 possibilités (1: fait sonner ou 2: ne fait pas sonner) et l'on fait passer 3 personnes ce qui correspond à une répétition identique et aléatoire. On peut donc modéliser la situation par une loi binomiale. \[ X \sim \mathcal{B}(3; 0.76) \] \item Probabilité qu'une seule personne fasse sonner le portique. On voit qu'il y a 3 branches qui correspondent à cette situation dont \[ P(X = 1) = 3 \times 0.68^1 \times 0.32^2 \approx 0.209 \] \item \[ P(X = 0) = 0.32^3 \approx 0.033 \] \[ P(X \geq 2) = P(X = 2) + P(X = 3) = 3 \times 0.68^2 \times 0.32^1 + 0.68^3 \approx 0.758 \] \item Il faut d'abord tracer le tableau résumant la loi de probabilité: \begin{center} \begin{tabular}{|c|*{4}{c|}} \hline Valeur & 0 & 1 & 2 & 3 \\ \hline Probabilité & $0.033$ & $0.209$ & $0.444$ &$0.314$ \\ \hline \end{tabular} \end{center} On peut alors calculer l'espérance \[ E[X] = 0 \times 0.033 + 1 \times 0.209 + 2 \times 0.444 + 3 \times 0.314 = 2.04 \] On peut donc estimer qu'il y aura en moyenne $2.04$ personnes qui feront sonner le portique sur les 3 personnes. \end{enumerate} \end{solution} \begin{exercise}[subtitle={Équation puissance}] Résoudre les équations et inéquations suivantes \begin{multicols}{2} \begin{enumerate} \item $10^x = 45$ \item $10^x = 5$ \item $0.69^x \leq 42$ \item $4 \times 0.04^x = 21$ \end{enumerate} \end{multicols} \end{exercise} \begin{solution} Les solutions ci-dessous ne sont pas justifiée car l'ordinateur ne sait pas faire. Par contre, vous vous devez savoir justifier vos réponses! \begin{enumerate} \item $x = \log(45)$ \item $x = \frac{\log(5)}{\log(10)}$ \item Il faut faire attention quand on divise par un log car ce dernier peut être négatif ce qui est le cas ici. Il faut donc pense à changer le sens de l'inégalité. $x \geq \frac{\log(42)}{\log(0.69)}$ \item Il faut penser à faire la division à par $4$ avant d'utiliser le log car sinon, on ne peut pas utiliser la formule $\log(a^n) = n\times \log(a)$. $x = \frac{\log(5.25)}{\log(0.04)}$ \end{enumerate} \end{solution} \begin{exercise}[subtitle={Étude de fonctions}] Soit $f(x) = 10x^3 - 645x^2 - 7200x - 16$ une fonction définie sur $\R$. \begin{enumerate} \item Calculer $f'(x)$ la dérivée de $f(x)$. \item Calculer $f'(48)$ et $f'(-5)$. \item En déduire une forme factorisée de $f'(x)$. \item Étudier le signe de $f'(x)$ et en déduire les variations de $f(x)$. \item Est-ce que la fonction $f(x)$ admet un maximum ou un minimum? Si oui, calculer sa valeur. \end{enumerate} \end{exercise} \begin{solution} \begin{enumerate} \item Dérivée de $f(x)$: $f'(x) = 30x^2 - 1290x - 7200$ \item \begin{align*} f'(48) &= 30 \times 48^{2} - 1290 \times 48 - 7200\\&= 30 \times 2304 - 61920 - 7200\\&= 69120 - 69120\\&= 0 \end{align*} \begin{align*} f'(-5) &= 30 \times - 5^{2} - 1290(- 5) - 7200\\&= 30 \times 25 + 6450 - 7200\\&= 750 - 750\\&= 0 \end{align*} Donc $x = 48$ et $x=-5$ sont des racines de $f'(x) = 30x^2 - 1290x - 7200$. \item On en déduit la forme factorisée suivante \[ f'(x) = 30 (x - 48)(x--5) \] \item Pas de correction disponible \item À causes des branches extérieurs, la fonction $f(x)$ n'a pas de maximum ou de minimum. \end{enumerate} \end{solution} %\printsolutionstype{exercise} \end{document} %%% Local Variables: %%% mode: latex %%% TeX-master: "master" %%% End: